Вы находитесь на странице: 1из 50

Test -03

1. Non-Resident Indians (NRIs) can invest in India as per a regulation of Foreign Exchange
Management (Transfer or issue of Security by a person resident outside India) Regulations,
2000. However, as per the regulation

1. NRIs cannot invest in shares or debentures.


2. NRIs may contribute foreign capital either by way of capital contribution

Which of the above is/are correct?


A. 1 only
B. 2 only
C. Both 1 and 2
D. None

Statement 1: As per Regulations, NRIs may invest in shares or partly paid up shares of an
Indian company under Foreign Direct Investment (FDI) Scheme subject to certain terms and
conditions.
Statement 2: NRIs may even acquire securities or units on a Stock Exchange in India on
repatriation basis (i.e. they can take the investment back abroad) under the Portfolio
Investment Scheme.
NRIs even hold FCNR deposits in India banks that earn them interest. We will cover FCNR in
details in coming tests.
Q Source: DSM/SBS/KA (Release ID :169208)

2. Hambantota deep-sea port was recently see in news because

a. Sri Lanka agreed to sell a strategic stake in this port to China.


b. A Major oil spill here was treated by genetically engineered bacteria.
c. India has signed a MoU with Seychelles to develop this port for defense related
purposes.
d. It will be used as an alternate transit to the Straits of Malacca.

Hambantota port is a deep-water port in the southern tip of Sri Lanka. It sits close to busy east-
west shipping lanes connecting Europe and Asia. Under the 99-year lease agreement, China
will invest over a billion dollars in the port and marine related activities for developing this
port.
Learning:
The port is expected to play a key role in China’s Belt and Road initiative (BRI), which
will link ports and roads between China and Europe. It is also considered that, this deal gives
an advantage to China in the bunkering business, which provides fuel to ships. India is
apprehensive that the port is part of Chinese ‘string of pearls’ with an objective to surround
India and dock its military vessels.
Q Source:
http://www.thehindu.com/news/international/sri-lanka-china-sign-11 -bn-hambantota-port-deal/article19385932.ece

IoI Page 1
http://www.thehindu.com/news/international/sri-lanka-china-sign-11 -bn-hambantota-port-deal/article19385932.ece

3. A ‘Task Force on Shell Companies’ under the Joint Chairmanship of Revenue Secretary and
Secretary, Ministry of Corporate Affairs was constituted in February, 2017. What are ‘Shell
Companies’

A. Oil companies that cause heavy ecological damage


B. Fly by night operators
C. A corporate entity without active business operations
D. A company engaged in deep sea precious stones extraction

Concept: Shell Company is a corporate entity without active business operations or significant
assets. It can’t be asserted that shell corporations are illegal. They are deliberate financial
arrangements by many big companies to avoid taxes without attracting legal actions.
Tax avoidance is not illegal, though it is not desirable.
Learning: But many shell companies park black money, carryout illegal transactions and
sometimes act as facilitators of money laundering. Often, shell companies remain untraceable
and happen to be the vehicle of choice for money launderers, bribe givers and takers, tax
evaders and financiers of terrorism.
Most of the shell companies are registered in tax havens like British Virgin Islands or Cayman
Islands. Recently it was found that a single address hosted 75 firms and no employees! The
taskforce

4. With reference to “flammable ice”, recently seen in news, consider the following statements:

1. It consists of methane trapped within water crystals.


2. It is another name for permafrost.
3. All of its reserves are trapped in plateaus at higher latitudes.
4. India is considering it as an alternative energy source.

Select the correct answer using the codes below.


A. 1, 3 and 4 only
B. 2 and 3 only
C. 1 and 4 only
D. only

Justification:
Statement 1:
Flammable ice (also known as methane hydrate or methane clathrates) consists of
methane trapped within water crystals. It is the world’s largest natural gas resource
trapped beneath permafrost and ocean sediment where low temperature and moderate
pressure combine to trap methane in this specific way.
Statement 2:
Permafrost appears on top of it. Permafrost is frozen chunks of ice that often contain
carbon and volatile gases.
Statement 3:
China has successfully produced natural gas from methane hydrate, also known as
“flammable ice”, in an experimental project in the South China Sea (SCS). Most of its
reserves are located deep in ocean floors, especially continental shelves.

IoI Page 2
reserves are located deep in ocean floors, especially continental shelves.
Statement 4:
India, USA, China and Canada all are looking at it as an alternative energy source.

Learning:
The methane hydrate is highly flammable and energy-intensive fuel as one cubic metre of
the compound can releases about 160 cubic metres of gas. It can break down into water
and methane after temperature is raised or pressure is lowered.
Q Source: China’s South China sea ocean drilling programme

5. The Government is implementing an Ethanol Blended Petrol (EBP) Programme in India.


Consider the following with reference to it.

1. The Government has introduced free market pricing mechanism for procurement of
ethanol under EBP Programme.
2. The Government has allowed procurement of ethanol produced from cellulosic and
lignocellulose feedstock.

Which of the above is/are correct?


A. 1 only
B. 2 only
C. Both 1 and 2
D. None

Background:
For the development of Biofuels, a National Policy on Biofuels has been announced in the
year 2009. So, under the EBP Oil Marketing Companies sell ethanol blended petrol with
percentage of ethanol up to 10% as per BIS specifications, depending upon its availability.

Justification:
Statement 1:
The prices are administered so that the availability of ethanol can be improved by
adequately incentivizing the producers.
Statement 2:
This is an eco-friendly way to convert ethanol from bio feed stock.
Learning:
In order to encourage production of Biodiesel in the country, the Government had also
announced the Biodiesel Purchase Policy in October, 2005. The Government has also
permitted the direct sale of Biodiesel (B-100) for blending with High Speed Diesel to all
consumers, in accordance with the specified blending limits and the standards specified
by the BIS.
Q Source: RM/RS- USQ1844 - LS (Release ID :169151)

6. With reference to Indian Green Building Council (IGBC), consider the following statements.

1. It is a body under Department of Urban Development.


2. It organizes the Green Building Congress.
3. It offers green building rating programmes.

IoI Page 3
Select the correct answer using the codes below.
A. 1 only
B. 2 and 3 only
C. 2 only
D. 3 only

Justification:
The IGBC is part of the Confederation of Indian Industry (CII) formed in 2001. It offers a
wide array of services including developing new green building rating programmes, green
building training programmes and certification services. It also organizes Green Building
Congress, its annual flagship event on green buildings. It closely works with several State
Governments, Central Government, World Green Building Council etc.
Q Source: IGBC conference 2017

7. Consider the following about Advance Pricing Agreements (APA) scheme

1. It was introduced by the Income-tax (IT) Act.


2. It cannot be signed with individual companies.
3. It endeavors to provide certainty to taxpayers in the domain of transfer pricing.
4. It is not applicable to international transactions.

Select the correct answer using the codes below.


A. 1 and 3 only
B. 2 and 4 only
C. 2, 3 and 4 only
D. 1, 3 and 4 only

Concept:
Suppose two subsidiaries A and B of the same company X are located in two different tax
regimes. Say A is subjected to more taxes, and B has to pay less tax. If company X
transfers most of its profit from A to B, it will need to less taxes then, because B is taxed
at lower rate. This is called transfer pricing, when profits are transferred to subsidiaries
to reduce tax burden.

Justification: Statement 2 and 3:


It is signed with companies so that this tax evasion can be regulated. It also gives
certainty to the tax department in terms of revenue collection.

Q Source: The Central Board of Direct Taxes (CBDT) has entered into 9 Unilateral Advance
Pricing Agreements (UAPAs) with Indian taxpayers in July, 2017.

8. Consider the following about Jiyo Parsi scheme.

1. It is a Central Sector Scheme focused on increasing employment opportunities for Parsis in


India.
2. It provides central assistance for expatriate Parsis to transfer their skills to native Parsis.

Which of the above is/are correct?


IoI Page 4
Which of the above is/are correct?
A. 1 only
B. 2 only
C. Both 1 and 2
D. None

Justification:
• Its main objective is to reverse the declining trend of Parsi population by adopting a
scientific protocol and structured interventions.
• It has two components:
○ Medical Assistance and
○ Advocacy (Counselling).
• The first phase of the scheme was launched initiated in 2013.
Learning:
• Minorites are defined both at the Central and State level. Parsis form a minority
community in India, and their welfare is the responsibility of the Central government.
• For a state declared minority, state governments provide for welfare plans.

Q Source: Ministry of Minority Affairs has launched Jiyo Parsi Publicity Phase-2 in Mumbai

9. ‘Mission Purpose’, ‘Breakthrough Starshot’ and ‘Sprites’ were recently seen in news in
context of

A. Interstellar missions
B. Cost effective sea water desalination plants
C. Autonomous self-driving cars
D. Genetically engineered clonal species

Learning:
• Starshot is a comprehensive space program launched under the US $100 million
Breakthrough Initiatives, announced by Yuri Milner and Stephen Hawking to develop and
launch practical interstellar space missions.
• The program aims to demonstrate proof of concept for light-propelled spacecraft that
could fly at 20 per cent of light speed.
• Its main objective is to send one-gram chips to star systems beyond the solar system in
search of extraterrestrial intelligence.
• Mission Purpose was designed to test the performance of the ‘Sprites’ (tiniest satellite)
electronics when in orbit.
• It will also explore the concept of solar sail propulsion in which spacecraft can be
powered using only the sun’s radiation.
• Sprites have been developed by researchers at Cornell University. Each of the mini Sprite
spacecrafts are built on a single 3.5*3.5 cm circuit board. They weigh just four grams
each.
• These tiny satellites also mark the next step in the field of spacecraft miniaturization that
can contribute to the development of centimeter and gram-scale StarChips envisioned
under the Breakthrough Starshot project.

Q Source: Often in news

IoI Page 5
10. Muntra, recently rolled out by the Defense Research and Development Organization (DRDO) is

A. Unmanned remotely operated tank


B. Stealth land mine
C. Indigenously developed fifth generation aircraft
D. Airborne Radar system for fighter planes

Learning:
It is India’s first unmanned, remotely operated tank at its Chennai lab. It can be used for
surveillance, mine detection and reconnaissance in areas with nuclear and biological
threats. It is also likely to be used in Naxal-hit areas.

Q Source: Defense related updates

11. ‘Sagar Vani’ system released by the Union Ministry of Earth Science is

a. An integrated information dissemination system that will serve the coastal


community regarding their safety
b. A SONAR system for deep sea mineral exploration
c. A language code for Indian Navy to communicate more securely
d. None of the above

Learning:
• It serves the coastal community, especially the fishermen community with the advisories
and alerts towards their livelihood as well as their safety at Sea.
• It is a software platform where various dissemination modes will be integrated on a
single central server.
• It includes Multi Lingual SMS, Voice Call/Audio Advisory, Social Media (Facebook, Twitter,
etc.) etc.
• Application:
○ It can disseminate services in local languages using advanced artificial intelligence
(AI) and machine learning capabilities.
○ This ‘Sagar Vani’ system compares with the most advanced countries’ services in
terms of speed of delivery, diverseness of services and omni channel capabilities.
○ The system also has facility to provide access to various stakeholders (State Fishery
Departments, Disaster Management Authorities, NGOs etc.) so that they will be
able to further disseminate these ocean information and alerts to the user
community.

Q Source: Initiative of Ministry of Earth Sciences: PIB

12. With reference to the World Monuments Fund (WMF), consider the following:

1. It is a dedicated fund under UNESCO managed by the affiliates of UNESCO.


2. Every member country of UNESCO has agreed to contribute to this fund.
3. Every year, WMF presents the Hadrian Award to international leaders who have advanced
the preservation of world architecture.

IoI Page 6
the preservation of world architecture.

Select the correct answer using the codes below.


A. 1 only
B. 3 only
C. 2 and 3 only
D. 2 only

Justification: Statement 1:
• WMF is a private, international, non-profit organization dedicated to the preservation of
historic architecture and cultural heritage sites around the world.
Statement 2:
• Founded in 1965, through donations and matching funds, WMF has worked with local
community and government partners worldwide to safeguard and conserve places of
historic value for future generations. To date, WMF has worked in more than 91
countries, including many UNESCO World Heritage Sites.
Statement 3:
• This applies both for preservation of world art and architecture.

Q Source: Additional Research:


http://www.thehindu.com/news/international/jesuss-tomb-to-be-unveiled-to-public/article17566017.ece

13. With reference to the technology, SurfNet, that was recently seen in news, consider the
following:

1. It is an artificial intelligence (AI) based software.


2. It can construct 3D models from 2D shapes.
3. It is presently being used for printing tissues and organelles of human beings.

Select the correct answer using the codes below.


A. 1 and 2 only
B. 2 and 3 only
C. 3 only
D. 2 only
Justification: Statement 1:
• SurfNet software utilizes machine learning to analyze 2D shapes and convert them into
projected 3D forms.
• In an advance version, it can allow future robots to navigate in the real world.
• In addition to transforming 2D shapes, the technology can also merge two shapes into
one another.
Statement 3:
• They were already being printed. Also, this is only a software, not a 3D printer. self-
driving cars. SurfNet will give them ability to understand the 3D environment around
them using standard 2D cameras. It can also be used to create 3D content for virtual
reality (VR) and augmented reality (AR) by simply using standard 2D photos.

Q Source: Breakthrough initiative has been in news for a long time

14. Who administers the oath of the President of India?

IoI Page 7
14. Who administers the oath of the President of India?
A. Vice-President
B. Prime Minister of India
C. Speaker, Lok Sabha
D. Chief Justice of India

Learning:
According to Article 60 of the Constitution of India, every President and every person
acting as President or discharging the functions of the President before entering upon
office shall make and subscribe oath to the office. The oath inter alia includes the oath to
preserve, protect and defend the constitution. In this sense, President takes oath as
guardian of Indian Constitution. The oath is administered by Chief Justice of India or in
his absence by the senior-most Judge of the Supreme Court available.

Q Source: Presidential Oath: July 2017

15. The Petrapole-Benapole Integrated Check Post is a major route for


A. India-Bangladesh bilateral trade
B. India-Bhutan border trade
C. India-China transit trade
D. India-Pakistan Wagha border trade

Learning:
• In order to facilitate movement of cargo across the border, India and Bangladesh have
recently agreed to operate Petrapole-Benapole ICP 24x7.
• The 24x7 operationalization of Petrapole-Benapole ICP is expected to be a significant
milestone towards expeditious clearance of cargo and, hence boost the bilateral trade
between the two countries.
• Land Ports Authority of India (LPAI) and Central Board of Excise and Customs (CBEC) are
the implementing agencies for this agreement from the Indian side.

Q Source: MJPS (Release ID :169332)

16. With reference to the Unorganised Workers' Social Security Act 2008, consider the following:
1. It covers self-employed workers as well as daily-wage workers.
2. It provides for constitution of National Social Security Board.
3. It issues a universal labour identification number (ULIN) to each labour in the
unorganized sector.
Select the correct answer using the codes below.
A. 1 and 2 only
B. 3 only
C. 2 and 3 only
D. 1 only

Justification: Statement 1:
• It is for the welfare of the unorganised workers (meaning home-based workers, self-
employed workers or daily-wage workers).
Statement 2:

IoI Page 8
Statement 2:
• The National Social Security Board at the Central level shall recommend formulation of
social security schemes viz life and disability cover, health and maternity benefits, old age
protection and any other benefit as may be determined by the Government for
unorganised workers.
Statement 3:
• LIN has been recently issued under Shramev Jayate Yojana since 2015. It wasn’t under
this act.

Q Source: AK (Release ID :169273)

17. With reference to manufacture of firecrackers in India, consider the following:

1. It is mandatory for firecracker manufacturers to mention the composition of explosives


and their quantity on the packet.
2. It is not mandatory to mention the consequential effects of fireworks on the environment
and human health on the label of the packet.

Which of the above is/are correct?


A. 1 only
B. 2 only
C. Both 1 and 2
D. None

Justification: Statement 1:
• This is as per the Explosives Rules, 2008.
Statement 2:
• However, it is not mandatory to mention the consequential effects of fireworks on the
environment and human health on the label of the packet or carton. This may be
because the effects are well known or that these products aren’t sold like general
consumer products.
• Their sale can be regulated by the district administration (for e.g. by providing an isolated
ground for sale of firecrackers).

Q Source: MJPS (Release ID :169309)

18. As per extant FDI policy in India

1. Foreign investment up to 100 percent is permitted for a company engaged in the business
of manufacturing mobile phones.
2. There is a 30 per cent local sourcing of component for any FDI related manufacturing
activity undertaken in India.

Which of the above is/are correct?


A. 1 only
B. 2 only
C. Both 1 and 2
D. None

IoI Page 9
Justification: Statement 1:
• This is under the automatic route where investment in the country can be made without
prior approval either of the Government or the Reserve Bank of India. FDI is largely a
matter of private business decisions, and therefore, no such details regarding efforts
made by foreign companies to set up for e.g. Apple i-phone facilities are required.

Statement 2:
• This isn’t applicable to all manufacturing units, for e.g. it is a restriction for mobile phone
manufacturing units in India.
• Recently Apple submitted a representation to waive off this requirement because it
would affect their profit margins.
• However, in case of ‘state-of-art’ and ‘cutting-edge technology’, sourcing norms can be
relaxed subject to Government approval. This relaxation will be valid for an initial period
of three years from the opening of first store, and thereafter, such entities will be
required to meet the domestic sourcing norms.

Q Source: MJPS (Release ID :169307)

19. Archaeological Sites are often preserved and cleaned using several chemical conservation
approaches by ASI. Which of these come under such approaches?
1. Using a mixture of moss and lichens on monuments
2. Adding Suspended Particulate Matter (SPM) to clean art pieces
3. Removal of calcareous deposits and accretions
4. Bio-cide treatment

Select the correct answer using the codes below.


A. 1, 2 and 3 only
B. 3 and 4 only
C. 2, 3 and 4 only
D. 1 and 4 only

Justification: Statement 1:
• There are certain biological factors such as Growth of Moss, Lichen, Algae, Fungi and
higher plants on a monument.
• They not only impart a ugly look to the monument but also cause physical & chemical
damaged to the building material. So, 1 is incorrect.
Statement 2:
• Suspended Particulate Matter (SPM) and other chemically active pollutant species along
with dust results in disfiguring of the monument. Similarly, variation in climatic
conditions, moisture and intense solar radiation are also responsible for the decay of
specific building materials.
Statement 3 and 4:
• General cleaning of monuments, mud pack cleaning, removal of calcareous deposits and
accretions, bio-cide treatment, consolidation and water repellent treatment, are some of
the known ways of chemical treatment.

Learning:
• Ancient Monuments and Archaeological Sites and Remains Act – 2010 provides for the

IoI Page 10
• Ancient Monuments and Archaeological Sites and Remains Act – 2010 provides for the
preservation of ancient and historical monuments, archaeological sites, and remains of
national importance and also provides penalty against acts of human vandalism. Other
methods of cleaning: Mud Pack Cleaning is used for the cleaning of plain and decorated
marble surface. This method is being used successfully for the conservation of Taj Mahal
and other marble structures. Very dilute mixture of ammonia solution and a non ionic
detergent is used for the easy removal of accretions and biological growth with mild
brushing. Bleaching powder slurry in aqueous medium is used to remove micro-
vegetation from lime plastered surface.

20. Consider the following about India Brand Equity Foundation (IBEF) and associated provisions.
1. It is a non-profit NGO certifying khadi and handloom products.
2. Its objective is to promote and create international awareness of the ‘Made in India’ label
in markets overseas.
3. India’s Foreign Trade Policy Statement released in 2015 emphasises the need of a
branding strategy.
4. Activities of IBEF are limited to plantations, textiles and leather sectors.

Select the correct answer using the codes below.


A. 2 and 3 only
B. 1 and 4 only
C. 1, 2 and 4 only
D. 1, 2 and 3 only

Justification: Statement 1:
• It is a trust established by the Department of Commerce, Ministry of Commerce and
Industry.
Statement 2:
• IBEF's primary objective is to promote and create international awareness of the Made
in India label in markets overseas and to facilitate dissemination of knowledge of Indian
products and services. Towards this objective, IBEF works closely with stakeholders
across government and industry
Statement 3:
• This is to promote Indian exports as well as push Make in India campaign.
Statement 4:
• IBEF has undertaken focused branding activities for sectors namely engineering, pharma,
plantations (tea, coffee and spices), services, textiles and leather.

Q Source: MJPS (Release ID :169316)

21. The Government has launched a new scheme namely “Financial Support to MSMEs in ZED
Certification Scheme”. The objective(s) of the scheme includes

1. Promote adaptation of energy efficient manufacturing


2. Drive manufacturing with adoption of Zero Defect production processes and without
impacting the environment.
3. Establish a MSME Product Certifying Authority (MPCA) to assess quality of their products
4. Support ‘Make in India’ campaign.

IoI Page 11
4. Support ‘Make in India’ campaign.

Select the correct answer using the codes below.


A. 1 and 3 only
B. 2 and 4 only
C. 1, 2 and 4 only
D. 1, 2, 3 and 4

Justification:
• ZED strives for quality manufacturing.
• The objective of the scheme for promotion of ZED manufacturing amongst MSMEs and
ZED Assessment for their certification so as to (these are other than what is mentioned in
the question): Develop an Ecosystem for Zero Defect Manufacturing in MSMEs.
Encourage MSMEs to constantly upgrade their quality standards in products and
processes. Develop professionals in the area of ZED manufacturing and certification.

Q Source: AK/RM (Release ID :169325)

22. Ministry of Tourism has released Guidelines for promotion of Medical and Wellness Tourism
as Niche Tourism Product. As per the guidelines
1. Ministry of Tourism offers financial support to accredited Medical and Wellness Tourism
Service Providers
2. Medical tourists from any country shall be provided Visa on arrival and e-visa facilities

Which of the above is/are correct?


a. 1 only
b. 2 only
c. Both 1 and 2
d. None

Justification:
• Tourism products including medical and wellness tourism are mostly driven by the
private sector and cost of such services are determined by market forces.
• However, the Ministry of Tourism is taking steps to promote this from the side of
government as well.
• The grant is for Marketing Development Assistance, for Publicity, organising
workshop/events/seminars and for organising Wellness and Medical Tourism Promotion
shows.

Q Source: Sanjay Kumar/jk/Parl. No. 1/26-07-2017: (Release ID :169061)

23. The VAJRA scheme was launched by the Science and Engineering Research Board (SERB) to
A. Connect the Indian academic and research and development (R&D) systems to the best of
global science and scientists
B. Develop anti-neurotoxins that can be used to neutralize agents used in a potential
chemical warfare
C. Provide mobile air-defence guards to marine ships used as warships
D. Increase participation from local ground level innovations to mainstream science and
technology
IoI Page 12
technology

Learning:
• SERB is a statutory body of the Department of Science and Technology (DST). It recently
launched a ‘Visiting Advanced Joint Research (VAJRA) Faculty Scheme’ for a sustained
international collaborative research.
• The scheme offers adjunct / visiting faculty assignments to overseas scientists, faculty
members and R&D professionals including NRI and OCI to undertake high quality
collaborative research in cutting edge areas of science and technology.
• This is to be done with one or more Indian collaborators of public funded academic and
research institutions of India.

Q Source: RDS/nb (Release ID :169081)

24. What is the importance of Kanchi in context of Ancient India?


1. Ghatika at Kanchi was an ancient centre of leaning attracting students from India and
abroad.
2. Founder of the Kadamba dynasty, Mayurasarman, studied Vedas at Kanchi.
3. Dharmapala, who later became the Head of the Ancient Nalanda University, belonged to
Kanchi.
4. It was known as the religious capital of the Southern India.
5. Kanchi hosts Kailasanatha which is one of the largest and most ornate ancient temples in
the whole of India.

Select the correct answer using the codes below.


A. 2 and 5 only
B. only
C. only
D. 1, 2, 3, 4 and 5

Justification:
• Once a capital of the Pallava dynasty, Kanchipuram was also a noted centre of learning
for Tamil and Sanskrit scholars. Dinganaga, a Buddhist writer came to study at Kanchi.
• It hosts many temples dedicated to Shiva and Vishnu.
• Kanchi was also the home of the famous 6th century CE poet Bharavi who wrote the
• Kiratarjuniya and the famous 11th to 12th century CE Hindu philosopher Ramanuja.
• Still today an important religious centre, the site has over hundred temples and is also
noted for its production of fine silk saris.

Q Source: Additional Research: Page 118: TN 11 th Standard Textbook

25. Pancha Siddhantika, a work of Varahamihira, deals with


A. Geographical domains of India
B. Astronomical systems
C. Occult and mysticism
D. Principles of social living

Learning:

IoI Page 13
Learning:
• He was also a great authority on astrology. In this context, he composed Pancha
Siddhantika, about the five astronomical systems. Along with Aryabhatta, his
contribution is considered seminal in Ancient India.
• His work Brihadsamhita is a great work in Sanskrit literature. It deals with a variety of
subjects like astronomy, astrology, geography, architecture, weather, animals, marriage
and omens. His Brihadjataka is considered to be a standard work on astrology.

Q Source: Page 108: TN 11 th Standard Textbook

26. In India, the safety aspects of genetically modified crops are assessed by which of these
bodies?
1. Institutional Biosafety Committees (IBSCs)
2. Review Committee on Genetic Manipulation (RCGM)
3. Genetic Engineering Appraisal Committee (GEAC)
4. National Biopiracy authority (NBA)
Select the correct answer using the codes below.
A. 1, 2 and 3 only
B. 2 and 4 only
C. 1 and 3 only
D. 2, 3 and 4 only

Background:
GEAC is the apex body constituted in the Ministry of Environment and Forests. It was set up
under 'Rules for Manufacture, Use, Import, Export and Storage of Hazardous
Microorganisms/Genetically Engineered Organisms or Cells 1989', under the Environment
Protection Act, 1986. The Rules of 1989 also define five competent authorities i.e. IBSC, RCGM,
GEAC, State Biotechnology Coordination Committee (SBCC) and District Level Committee (DLC)
for handling of various aspects of the rules.

Justification: Statement 1:
• Its main function is to note, examine and approve proposals involving rDNA work; to
ensure adherence of r-DNA Safety Guidelines- 1990 of Government and inspection of
containment facilities at R&D and production units.
Statement 2:
• Main functions of RGCM are:
○ To bring out manuals of guidelines specifying producers for regulatory process on
GMOs in research, use and applications including in industry with a view to ensure
environmental safety.
○ To review all on going r-DNA projects involving high risk category and controlled
field experiments
○ To lay down producers for restriction or prohibition, production, sale, import & use
of GMOs both for research and applications.
○ To authorize imports of GMOs/ transgenes/ transgenic seeds for research purposes.

Q Source: RDS/nb (Release ID :169320)

27. With reference to Ancient India, who introduced the Pattini cult in Tamilnadu?

IoI Page 14
27. With reference to Ancient India, who introduced the Pattini cult in Tamilnadu?
A. Cheran Senguttuvan
B. Rajaraja II
C. Pulakesin I
D. Narasinghvarmana

Justification:
• He belonged to 2nd century A.D.
• Senguttuvan introduced the Pattini cult or the worship of Kannagi as the ideal wife in
Tamil Nadu.
• The stone for making the idol of Kannagi was brought by him after his Himalayan
expedition.
• The consecration ceremony was attended by many princes including Gajabhagu II from
Sri Lanka.
Learning:
• His younger brother was Elango Adigal, the author of Silappathigaram.
• Among his military achievements, his expedition to the Himalayas was remarkable. He
defeated many north Indian monarchs.

Q Source: Page 87: TN 11 th Standard Textbook

28. Family Participatory Care (FPC) as an important concept of health care mainly focuses on
A. Newborn healthcare
B. Geriatric population
C. Urban sanitation and hygiene
D. Health insurance in India
Learning
• Sick and newborn are highly vulnerable and require careful nurturing in order to survive
the neonatal period and first year of life.
• This approach provides for partnership between health care staff and families in care of
sick newborns admitted in the Special Newborn Care Units (SNCU).
• Under it, the capacities of parents-attendants are built in newborn care through a
structured training programme.
• MoHFW has recently released Operational Guidelines on FPC. The guidelines will be
shared with the states for implementation to further improve the quality of care
provided in the SNCUs across the country.

Q Source: http://pib.nic.in/newsite/PrintRelease.aspx?relid=169187

29. The NITI Aayog has approved several high-tech mas rapid transport proposals to improve
public transport. Consider the following about them.
1. Hyperloop is a high air pressure tube connecting cities at speeds matching that of an
aircraft.
2. Freight rail roads will be elevated corridors with rail lines where freight trucks can move
on rails at high speed.
3. Pod taxis are underground transit systems that run entirely on solar power.
4. Hybrid electric buses combine a conventional internal combustion engine with an electric
propulsion system.

IoI Page 15
Select the correct answer using the codes below.
A. 1 and 3 only
B. 2 and 4 only
C. 2, 3 and 4 only
D. 1 and 2 only

Justification:
• This proposal includes high-tech mass rapid transportation technologies such as metrino,
hyperloop, pod taxis, stadler buses, hybrid buses and freight rail road.

Statement 1: They are near-vacuum tubes so that air friction in travel can be reduced.
Statement 2: A dedicated corridor allows for greater efficiency and speed.
Statement 3: These are usually cabled taxis running on an elevated drive train.
Statement 4:The addition of a battery-powered electric motor increases the fuel efficiency of
hybrids in a number of ways. Idle-off feature turns off the car's conventional engine when the
vehicle is stopped, saving fuel. The battery provides energy for the air conditioner and
accessories while the vehicle idles at stoplights or in traffic, and the electric motor can start
the vehicle moving again. If needed, the conventional engine will reengage to provide more
power for acceleration. Learning: "Regenerative braking" is another fuel-saving feature.
Conventional cars rely entirely on friction brakes to slow down, dissipating the vehicle's kinetic
energy as heat. Regenerative braking allows some of that energy to be captured, turned into
electricity, and stored in the batteries. This stored electricity can later be used to run the
motor and accelerate the vehicle.
Q Source:
http://www.livemint.com/Politics/8fnjxsUr13noU98kvkPerO/Transport-ministry-to-explore-6-new-ma
ss-rapid-transportatio.html

30. Aadhaar is presently being used as an identifier in respect of following school education
related schemes ?
1. Mid-Day Meal Scheme (MDMS)
2. Sarva Shiksha Abhiyan (SSA)
3. Rashtriya Madhyamik Shiksha Abhiyan (RMSA)
4. Inclusive Education for Disabled at Secondary Stage (IEDSS)
Select the correct answer using the codes below.
A. 1 and 2 only
B. 2, 3 and 4 only
C. 3 and 4 only
D. 1, 2, 3 and 4

Correct Answer : D

Justification:
• This is to reduce ghost beneficiaries, leakages in distribution of provisions. Also, this
enforces accountability, transparency and efficiency in the governance of the schemes.
Other covered schemes are:
• National Scheme of Incentive to Girls for Secondary Education (NSIGSE)
• National Means-cum-Merit Scholarship Scheme (NMMSS)
• Centrally Sponsored Scheme for providing quality education in Madrasa (SPQEM)

IoI Page 16
• Centrally Sponsored Scheme for providing quality education in Madrasa (SPQEM)

Q Source: GG/AK/RK/Aadhaar for Social Welfare: (Release ID :169280)

31. The Satavahanas patronized Buddhism and Brahmanism. The evidence of this is clear from
the fact that
1. They built several chaityas and viharas.
2. They prohibited Buddhist monks from acquiring land to help in their spiritual pursuit.
3. They persecuted Brahmanism.
4. They patronized the Prakrit language and literature.
Select the correct answer using the codes below.
A. 1, 2 and 3 only
B. 1 and 4 only
C. 1, 3 and 4 only
D. 2 and 3 only

Correct Answer : B

Justification: Statement 1:
• Vashishtaputra Pulamayi repaired the old Amaravathi stupa. Their architecture in
Nagarjunakonda was also notable.
Statement 2: They also made grants of villages and lands to Buddhist monks.
Statement 3: Brahmanism was revived by the Satavahanas along with the performance of
asvamedha and rajasuya sacrifices.
Statement 4: Hala’s Sattasai is an excellent piece of Prakrit literature.

Q Source: Page 77: TN 11 th Standard Textbook

32. With reference to Ancient India, in the northern part of the country, the village headman was
known as the
A. Gramabhojaka
B. Agrahara
C. Dhammsurna
D. Sentokwara

Correct Answer : A

Learning:
• Usually, men from the same family held the position for generations or via hereditary.
• The grama bhojaka was often the largest landowner. Generally, he had slaves and hired
workers to cultivate the land.
• Besides, as he was powerful, the king often used him to collect taxes from the village.
• He also functioned as a judge, and sometimes as a policeman.
• Apart from the gramabhojaka, there were other independent farmers, known as
grihapatis, most of whom were smaller landowners.

Q Source: Revision Previous tests: Chapter 7: 6th NCERT History

IoI Page 17
33. Sangam poems mention the term muvendar. It was used for
A. Land allotted to temples by Kings
B. Administrative arrangements made to felicitate scholars of Tamil
C. Heads of three ruling dynasties of Southern India
D. Classical commentary on Vedas

Correct Answer : C

Learning:
• This is a Tamil word meaning three chiefs, used for the heads of three ruling families, the
Cholas, Cheras, and Pandyas, who became powerful in south India around 2300 years
ago.
• Each of the three chiefs had two centres of power: one inland, and one on the coast.Of
these six cities, two were very important: Puhar or Kaveripattinam, the port of the
Cholas, and Madurai, the capital of the Pandyas.

Q Source: Additional Research: Page 139: TN 11 th Standard Textbook

34. Which of these developments can be attributed to Later Vedic age, as against the practice in
the Vedic age?
1. Women lost their political rights of attending assemblies.
2. Child marriage were banned and the practice of sati was absent.
3. Women were not allowed to wear garments made of cotton.
Select the correct answer using the codes below.
A. 1 only
B. 2 and 3 only
C. 3 only
D. 2 only

Correct Answer : A
Background:
• The Rig Vedic society was patriarchal. But, the condition of women was much better in
this period that in the Later vedic age.
• Women were given equal opportunities as men for their spiritual and intellectual
development. There were women poets like Apala, Viswavara, Ghosa and Lopamudra
during the Rig Vedic period.
• Women could even attend the popular assemblies. There was no child marriage and the
practice of sati was absent.
Justification: Statement 1:
• In the family, the power of the father increased during the Later Vedic period.
• There was no improvement in the status of women. They were still considered inferior
and subordinate to men. Women also lost their political rights of attending assemblies.
Statement 2:
• Child marriages had become common during the later vedic period.
Statement 3:
• In both periods, men and women wore upper and lower garments made of cotton and
wool. A variety of ornaments were used by both men and women.

IoI Page 18
Q Source: Revision Previous tests: Page 29: TN 11 th Standard Textbook

35. With reference to Ancient texts, consider the following statements.


1. The Brahmanas are the law texts under Manusmriti that helped establish the
predominance of Brahmana community.
2. Upanishads are treatises relating to prayer and sacrificial ceremony.
3. Aranyakas are called forest books and they deal with mysticism.
4. Yajurveda deals with music, chanting and medicine.
Select the correct answer using the codes below.
A. 1 and 2 only
B. 3 only
C. 1, 3 and 4 only
D. 2 and 4 only

Correct Answer : B

Justification:
• Besides the Vedas, there are other sacred works like the Brahmana, the Upanishads, the
Aranyakas and the epics Ramayana and Mahabharata. The Brahmanas are the treatises
relating to prayer and sacrificial ceremony.
• The Upanishads are philosophical texts dealing with topic like the soul, the absolute, the
origin of the world and the mysteries of nature.
• The Aranyakas are called forest books and they deal with mysticism, rites, rituals and
sacrifices. Yajurveda is a compilation of ritual offering formulas that were said by a priest
while an individual performed ritual actions such as those before the yajna fire.

Q Source: Revision Previous tests: Page 28: TN 11 th Standard Textbook

36. Irula people


1. Are a tribe of the Andaman and Nicobar Islands
2. Practice iron smelting as the main occupation
3. Do not speak any known dialect
Select the correct answer using the codes below.
A. 1 and 2 only
B. 3 only
C. 2 only
D. None of the above

Correct Answer : D

Justification: Statement 1:
• Irula is an ethnic group of India. They inhabit the area of the Nilgiri mountains, in the
states of Tamil Nadu and Kerala.
Statement 2:
• Traditionally, the main occupation of the Irulas has been snake and rat catching. They
also work as labourers (coolies) in the fields of the landlords during the sowing and
harvesting seasons or in the rice mills. Fishing is also a major occupation.
Statement 3:
IoI Page 19
Statement 3:
• People of Irula ethnicity are called Irular, and speak Irula, which belongs to the Dravidian
family. Unlike the Negrito tribes in the Andaman Islands who have retained their
language, Irular speak the Irula language, a Dravidian language that is closely related to
Tamil, Yerukala, Sholaga and other Tamil languages.

Q Source: Additional Research: Page 90: TN 11 th Standard Textbook

37. Buddhamitra was known for


1. Writing virasoliyam which is a work on Tamil Grammar
2. Erecting inscriptions on images of bodhisattvas and the Buddha near Ganges river
Which of the above is/are correct?
A. 1 only
B. 2 only
C. Both 1 and 2
D. None

Correct Answer : C

Justification:
• Buddhamitra was a Buddhist nun from India during the Kushan Empire.
Statement 1:
• She is remembered because of dated inscriptions on images of bodhisattvas and the
Buddha that she erected in three cities near the Ganges river. They mark her success in
attracting money and patronage to the Sarvastivada, the sect of Buddhism to which she
belonged.
Statement 2:
• Virasoliyam attempts to find synthesis between Sanskrit and Tamil grammar. There were
other books written on Tamil grammar as well like Yapperungalam and
Yapperungalakkarigai by the Jain ascetic Amirtasagara.

Q Source: Additional Research: Page 150: TN 11 th Standard Textbook

38. Adipurana written by Gunabhadra is a work on the


A. Commentary of esoteric aspects found in the Vedas
B. Life stories of various Jain saints
C. Sanskrit Grammar
D. Verses used in sacrificial ceremony

Correct Answer : B

Justification:
• It is a 9th century Sanskrit poem composed by Jinasena, a Digambara monk. It deals with
the life of Rishabhanatha, the first Tirthankara, and other saints.
Learning:
• Amogavarsha I, who was a Jain patronized many Jain scholars. His teacher Jinasena
composed Parsvabhudaya, a biography of Parsva in verses. Sakatayana wrote the
grammer work called Amogavritti. And, the great mathematician of this period,
Viracharya was the author of Ganitasaram.
IoI Page 20
Viracharya was the author of Ganitasaram.

Q Source: Page 137: TN 11 th Standard Textbook

39. The images of Shiva represented in various forms like Nataraja, Gangadhara,
Ardhanareesvara, Somaskanda and Trimurthi can be found in
A. Elephanta caves
B. Ajanta caves
C. Karla caves
D. Kanheri rock-cut caves

Correct Answer : A
Learning:
• Elephanta is an island near Bombay. It was originally called Sripuri. The Portuguese after
seeing the large figure of an elephant named it Elephanta. The sculptural art of the
Rashtrakutas reached its zenith in this place.
• There is a close similarity between the sculptures at Ellora and those in Elephanta. They
might have been carved by the same craftsmen.
• The most imposing figure of this temple is Trimurthi. The sculpture is six metre high. It is
said to represent the three aspects of Shiva as Creator, Preserver and Destroyer.

Q Source: Page 138: TN 11 th Standard Textbook

40. If you compare the socio-economic life of Cholas and Sangam age, what is/are the chief
similarities in these periods with respect to the position of women in society?
1. The practice of Sati was present.
2. Devadasi system was abolished.
3. Women were not allowed to write or study literature.
4. Love marriage was forbidden.
Select the correct answer using the codes below.
A. 1 only
B. 2, 3 and 4 only
C. 3 and 4 only
D. 1, 2 and 4 only

Correct Answer : A
Justification: Statement 1 and 4:
• Love marriage was a common practice in the Sangam age. Women were allowed to
choose their life partners. However, the life of widows was miserable. The practice of Sati
was prevalent in the higher strata of society. Even in Cholas, practice of ‘sati’ was
prevalent among the royal families.
Statement 2:
• The devadasi system or dancing girls attached to temples emerged during the Chola
period. There is evidence that they were often subjected to harassment by priets and
officials.
Statement 3:
• Women poets like Avvaiyar, Nachchellaiyar, and Kakkaipadiniyar flourished in the
sangam period and contributed to Tamil literature.

IoI Page 21
Q Source: Page 91 and 149: TN 11 th Standard Textbook

41. With reference to Chola Education and Literature, consider the following:
1. Temples and mathas served as the only educational centres.
2. Not only Vedas and Epics, but subjects like mathematics and medicine were also taught.
3. Endowment of lands was made to run educational centres.
Select the correct answer using the codes below.
A. 1 and 2 only
B. 2 and 3 only
C. 3 only
D. 1, 2 and 3

Correct Answer : B

Justification: Statement 1:
• During this period, besides the temples and mathas as educational centres, several
educational institutions also flourished. The inscription at Ennayiram, Thirumukkudal and
Thirubhuvanai provide details of the colleges existed in these places.
Statement 2 and 3:
• The development of Tamil literature reached its peak during the Chola period. And a lot
of it came from the contribution of education and literature to the enrichment of this
culture.
• Mathematics and medicine studies showed that Kings encouraged logical and intellectual
pursuits and were not centred entirely on religious studies.

Q Source: Page 149: TN 11 th Standard Textbook

42. Vanavaramban, Vanavan, Kuttuvan, Irumporai and Villavar were famous titles assumed by
A. Chera kings
B. Sangam poets
C. Thiruvalluvar, a tamil philosopher and poet
D. Temple priests in Southern Indian dynasties

Correct Answer : A

Learning:
• Hereditary monarchy was the form of government during the Sangam period. The Chera
kings assumed titles like Vanavaramban, Vanavan, Kuttuvan, Irumporai and Villavar, the
Chola kings like Senni, Valavan and Killi and the Pandya kings Thennavar and Minavar.
• Each of the Sangam dynasties had a royal emblem – carp for the Pandyas, tiger for the
Cholas and bow for the Cheras.

Q Source: Page 89: TN 11 th Standard Textbook

43. Consider the following statements with reference to the South Indian dynasties.
1. Arikkamedu was a seaport during the Sangam period.
2. Uraiyur was famous for pearls.

IoI Page 22
2. Uraiyur was famous for pearls.
3. Lord Murugan was the primary deity of the Sangam age.
Select the correct answer using the codes below.
A. 1 only
B. 2 and 3 only
C. 3 only
D. 1, 2 and 3

Correct Answer : D

Justification: Statement 1:
• External trade was carried between South India and the Greek kingdoms.
• The port city of Puhar became an emporium of foreign trade, as big ships entered this
port with precious goods.
• Other ports of commercial activity include Tondi, Musiri, Korkai, Arikkamedu and
Marakkanam. Statement 2: Uraiyur was an ancient Chola city with a fortress and city wall
on the southern banks of the river Kaveri. It was known for pearls.
Statement 3: The worship of Murugan was having an ancient origin and the festivals relating
to God Murugan was mentioned in the Sangam literature. He was honoured with six abodes
known as Arupadai Veedu.

Q Source: Page 92: TN 11 th Standard Textbook

44. Consider the following about the visit of the famous Chinese traveller, Fahien.
1. He visited India during the reign of Chandragupta II.
2. He stayed for over twenty years in India.
3. He came to India by sea route.
4. His main purpose of visiting India was to study its political and social system.
5. He studied Sanskrit in Patliputra.
Select the correct answer using the codes below.
A. 2, 3 and 4 only
B. 1, 2 and 5 only
C. 1 and 5 only
D. 1, 2, 4 and 5 only

Correct Answer : C

Justification: Statement 1 and 2:


• Out of his nine years stay in India, he spent six years in the Gupta empire.
Statement 3:
• He came to India by the land route through Khotan, Kashgar, Gandhara and Punjab.
• He visited Peshawar, Mathura, Kanauj, Sravasti, Kapilavastu, Kusinagara, Pataliputra, Kasi
and Bodh Gaya among other places. He returned by the sea route, visiting on the way
Ceylon and Java.
Statement 4:
• The main purpose of his visit was to see the land of the Buddha and to collect Buddhist
manuscripts from India.
Statement 5:
• He stayed in Pataliputra for three years studying Sanskrit and copying Buddhist texts.

IoI Page 23
• He stayed in Pataliputra for three years studying Sanskrit and copying Buddhist texts.

Q Source: Page 102: TN 11 th Standard Textbook

45. Hiuen Tsang portray the social life in the times of Harsha. Consider the following with
eference to it.
1. He mentions that the Sudras were forbidden from agriculture.
2. As per his documentation, the dead could not be subjected to water burial.
3. Dowry system was practiced in the empire.
Select the correct answer using the codes below.
A. 1 only
B. 2 and 3 only
C. 3 only
D. 1 and 2 only

Correct Answer : C

Justification:
• The fourfold division of the society – Brahmin, Kshatriya, Vysya and Sudra – was
prevalent.
Statement 1:
• The Brahmins were the privileged section of the society and they were given land grants
by the kings. Hiuen Tsang mentions that the Sudras practiced agriculture.
Statement 2:
• Hiuen Tsang mentions three ways of disposal of the dead – cremation, water burial and
exposure in the woods.
Statement 3:
• The position of women was not satisfactory. The institution of Swyamvara (the choice of
choosing her husband) had declined. Remarriage of widows was not permitted,
particularly among the higher castes. The system of dowry had also become common.
The practice of sati was also prevalent.
Learning:
• Trade and commerce had declined during Harsha’s period, which is is evident from the
decline of trade centres, less number of coins, and slow activities of merchant guilds.
• The decline of trade in turn affected the handicrafts industry and agriculture.
• Since there was no large-scale demand for goods, the farmers began to produce only in a
limited way. This led to the rise of self-sufficient village economy.

Q Source: Page 117: TN 11 th Standard Textbook

46. IFAW (International Fund for Animal Welfare)


1. is a non-profit organization
2. provides hands-on assistance to animals
3. campaigns to end commercial whaling
4. was founded by the World Wildlife Fund (WWF)
Select the correct answer using the codes below.
A. 1 and 4 only
B. 2 and 3 only
C. 1, 2 and 3 only

IoI Page 24
C. 1, 2 and 3 only
D. 2 and 4 only

Correct Answer : C

Justification:
• The International Fund for Animal Welfare (IFAW) is one of the largest animal welfare
and conservation charities in the world.
• The group's declared mission is to "rescue and protect animals around the world.
• The International Fund for Animal Welfare (IFAW) was founded by a small group of
people in 1969, to stop the commercial hunt for seal pups in Canada.
• The International Tiger Day (also known as Global Tiger Day) is celebrated every year on
29 July to raise awareness for tiger conservation.
• The IFAW recently organized major events on International Tiger conservation day.
You can read more about it here http://www.ifaw.org/united-states/about-ifaw

47. Consider the following:


1. They developed Vesara style of architecture.
2. Their structural temples exist at Aihole, Badami and Pattadakal.
3. Their administration was highly centralized.
The above most accurately refer to?
A. Cholas
B. Chalukyas
C. Cheras
D. Pandyas

Correct Answer : B

Administration:
• The Chalukya administration was highly centralized unlike that of the Pallavas and the
Cholas. Village autonomy was absent under the Chalukyas.
Architecture:
• The Chalukyas were great patrons of art. They developed the vesara style in the building
of structural temples. However, the vesara style reached its culmination only under the
Rashtrakutas and the Hoysalas. Cave temple architecture was also famous under the
Chalukyas. Their cave temples are found in Ajanta, Ellora and Nasik. The best specimens
of Chalukya paintings can be seen in the Badami.

Q Source: Additional Research: Page 134: 11 thTN Textbook

48. With reference to Ancient India, mahajanapada rulers imposed taxes on


A. Crops and crafts persons only
B. Goods, crops, herders, hunter gatherers and crafts persons
C. Goods and royal services only
D. Hunter gatherers and crafts persons only

Correct Answer : B

IoI Page 25
Background:
• As they were building huge forts and maintaining big armies, they needed more
resources. Instead of depending on occasional gifts brought by people, as in the case of
the raja of the janapadas, they started collecting regular taxes.
Learning:
• Taxes on crops were the most important. This was because most people were farmers.
There were taxes on crafts persons as well. These could have been in the form of labour.
For example, a weaver or a smith may have had to work for a day every month for the
king. Herders were also expected to pay taxes in the form of animals and animal produce.

• There were also taxes on goods that were bought and sold, through trade.
• And hunters and gatherers also had to provide forest produce to the raja.

Q Source: Revision previous tests: Page 59-60: 6th NCERT

49. Dakshinapatha and Uttarapatha were


A. Two great highways that have connected different parts of the sub-continent since the
iron Age
B. Pali transcriptions of two different sects of Buddhism
C. Extensions of Chola Empire in Sri Lanka and Northern India respectively
D. The largest gopurams of Brihadeshwara temple made under Rajaraja

Correct Answer : A

Learning:
• Dakshinapatha is a historical which has been used to describe the great southern
highway in India, traveling from Magadha to Pratishthana.
• Uttarapatha or the great northern road ran from Taxila in Afganisthan, through the
modern Punjab up to the western coast of Yamuna.
• Following the course of Yamuna, it went southwards up to Mathura, from there it passed
on to Ujjain in Malwa and to Broach on western coast.
• The most important ruler of the Satavahanas, Gautamiputra Shri Satakarni and other
Satavahana rulers were known as lords of the dakshinapatha.

Q Source: Additional Research: Page 99: 11 thTN Textbook

50. Consider the following about Traditional Knowledge Digital Library (TKDL), an Indian
initiative.
1. It is to protect Indian traditional knowledge from exploitation by wrongful patents mainly
at International Patent Offices.
2. It is maintained in a digitized format and is available only in old Sanskrit so as to avoid the
misuse of ancient discoveries.
Which of the above is/are correct?
A. 1 only
B. 2 only
C. Both 1 and 2
D. None

Correct Answer : A
IoI Page 26
Correct Answer : A

Justification: It is an organisation of the Council of Scientific and Industrial Research.


TKDL contains Indian traditional medicine knowledge available in public domain and pertains
to classical/ traditional books related to Ayurveda, Unani and Siddha in a digitized format and
is available in five international languages (English, French, German, Spanish and Japanese).
India’s rich and time-tested traditional medicinal knowledge which exists in languages such as
Sanskrit, Hindi, Arabic, Persian, Urdu, Tamil etc. is neither accessible nor comprehensible for
patent examiners at the international patent offices. So, let’s say if some corporation patents
the use of termuric in curing certain chronic diseases, it’ll amount to piracy of our traditional
knowledge. If this information about ancient knowledge is already made available to patent
offices for pre verification in different languages beforehand, such misuse can be stopped.

51. Consider the following about the Senior Citizens Welfare Fund.
1. It was established along with the Central Social Welfare Board (CSWB).
2. It will be administered by an Inter-Ministerial Committee.
3. The fund shall not be audited by the CAG due to its fundamental character as a welfare
fund.
Select the correct answer using the codes below.
A. 2 only
B. 1 and 3 only
C. 1 and 2 only
D. 2 and 3 only

Correct Answer : A

Justification: Statement 1:
• CSWF was established in the 1950s, so 1 is wrong. You can read about CSWB
here http://www.cswb.gov.in/index1.asp?linkid=230&langid=1The
• Centre brought in Senior Citizens’ Welfare Fund Act, 2015 (SCWF) as part of the
Finance Act, 2015, which mandates transfer of unclaimed amounts of
policyholders to the fund (SCWF) after a period of 10 years.
Statement 2:
• The fund will be administered by an Inter-Ministerial Committee, headed by a
Chairperson. The Committee will be competent to spend money from the fund
for satisfying various objectives.
Statement 3:
• The accounts of the fund will be open to audit by CAG, regularly. The Central
Government will present the annual report and the one furnished by CAG to be
laid before the Parliament.

52. National Anti-profiteering Authority will look after


A. Enforcing anti-trust or competitive laws in the industry on the lines of Competition
Commission of India (CCI)
B. Consumer grievances arising due to complaints raised by unreasonable pricing of goods
and services
C. Ensuring that benefits of a reduction in tax on supply of goods or services flow to the
Consumer
D. None of the above
IoI Page 27
D. None of the above

Correct Answer : C

Concept
• In the GST mechanism, the producers are provided an input tax credit. It works this way.
• Suppose, a producer requires INR 80 worth of cotton to produce a shirt, which is finally
processed and priced at INR 100 in the market.
• In this process, the producer will have to pay taxes at two places – a) while purchasing
the raw material, and b) while selling the final produce – shirt.
• This double taxation (tax on tax) is avoided by an input tax credit mechanism. Under this,
the producers are compensated for any additional taxes they would have paid.
Justification:
• When constituted by the GST Council, the National Anti-profiteering Authority shall be
responsible for applying anti-profiteering measures in the event of a reduction in rate of
GST on supply of goods or services.
• It will also check if the benefit of input tax credit is not passed on to the recipients by way
of commensurate reduction in prices.
Learning:
• In the event the National Anti-profiteering Authority confirms the necessity of applying
anti-profiteering measures, it has the power to order the business concerned to reduce
its prices or return the undue benefit availed along with interest to the recipient of the
goods or services. If the undue benefit cannot be passed on to the recipient, it can be
ordered to be deposited in the Consumer Welfare Fund. In extreme cases the National
Anti-profiteering Authority can impose a penalty on the defaulting business entity and
even order the cancellation of its registration under GST.

Q Source: http://pib.nic.in/newsite/PrintRelease.aspx?relid=169000

53. Consider the following about electoral bonds that were introduced in the 216-17 annual
budget.
1. They will be an interest-paying debt instrument.
2. They cannot be sold by banks.
3. They can be deposited in notified accounts of political parties.
4. The bonds will not carry the name of the donor.
Select the correct answer using the codes below.
A. 1 and 2 only
B. 3 and 4 only
C. 1, 2 and 3 only
D. 2 and 3 only

Correct Answer : B

Concept:
• Electoral bonds are to be used for donations to political parties. The advantage of the
electoral bond system over donations through cheques is that many donors expressed
reluctance to use cheques because it becomes transparent and leads to political vendetta
by rivals.
Features:

IoI Page 28
Features:
• As per the electoral bond mechanism announced in the Budget, the proposed bonds will
resemble a promissory note and not an interest-paying debt instrument.
• They will be sold by authorised banks and can be deposited in notified accounts of
political parties within the duration of their validity.
• The bonds will not carry the name of the donor and routing of the money through banks
will ensure that only tax paid money comes into the political system.
• The government is planning to go ahead with electoral bonds even if consensus eludes
on the issue as political parties have not come up yet with any suggestion on the
proposal.
Q Source: http://www.insightsonindia.com/2017/07/25/insights-daily-current-affairs-25-july-2017/

54. Anti-dumping duty is usually applied in order to


1. Contain trade distortion
2. Increase imports of capital goods
3. Sterilize foreign capital inflows
Select the correct answer using the codes below.
A. 1 and 2 only
B. 3 only
C. 1 only
D. 2 and 3 only

Correct Answer : C

Justification: Statement 1:
• Suppose a nation like China is dumping (exporting) its goods at exceptionally cheap rice's
so that it can capture the market. Since these cheap prices can’t be justified by market
forces alone, the Indian government must apply a duty (like a tax) on the product to raise
its price. This is to shield the domestic producers.
Statement 2:
• Capital goods are usually not dumped by nations, since the industry is highly segmented
and its hard to dump products at cheap prices.
Statement 3:
• It has nothing to do with FDI or FII.
Learning:
• India has initiated over two hundred anti-dumping investigations between 2012 and
2017 against various countries, including China and Indonesia. During the period,
maximum number of cases were against China. Product categories on which the levy was
imposed include chemicals, fibre boards, glass & glassware, pharmaceuticals and steel.

Q Source: http://www.insightsonindia.com/2017/07/25/insights-daily-current-affairs-25-july-2017/

55. If China were to takeover Doklam plateau, which region in or near India would be most
affected by this move?
A. Leh and Ladakh
B. Siliguri Corridor
C. India-Uttarakhand Border
D. Sundarban delta

IoI Page 29
Correct Answer : B

Learning:
• China is building a road project in the region which as per Bhutan would run from the
town of Dokola to the Bhutanese army camp at Zompelri.
• This is seen by Bhutan as a direct violation of the agrements between two nations.
• India has said the Chinese road project threatens its access to the Siliguri corridor that
connects our mainland to north-eastern states.
• The tense standoff between India and China over the Doklam plateau has only escalated,
raising concerns in both capitals of an all-out military conflict.
Q Source: http://www.insightsonindia.com/2017/07/25/insights-daily-current-affairs-25-
july-2017/

56. The Mekedatu dispute recently seen in news is between


A. Karnataka and Tamil Nadu
B. Government of India and Government of Sri Lanka
C. Fishermen community from Kerala and Northern province of Sri Lanka
D. Andhra Pradesh, Telangana and Karnataka

Correct Answer : A

Learning:
• Karnataka intends to build a reservoir across river Cauvery near Mekedatu in Kanakapura
taluk.
• It was first proposed along with Shivanasamudra hydro power project in 2003 with an
intention to use the water for a hydro power station and supply drinking water to
Bengaluru city.
• However, Tamil Nadu objected saying Karnataka had not sought prior permission for the
project. Its argument was that the project would affect the flow of Cauvery water to
Tamil
: http://www.insightsonindia.com/2017/07/26/insights-daily-current-affairs-26-july-2017/

57. Why the government has introduced the Central Road Fund (Amendment) Bill, 2017 in the
Lok Sabha?
A. To provide for development of national waterways (NWs) from this fund
B. To add state and local highways to the provision list of this fund
C. To upgrade road construction and maintenance in the category of plan expenditure
D. To raise revenue for road development via foreign direct investment

Correct Answer : A

Background:
• The fund is utilised for the development and maintenance of National highways, State
roads, Rural roads and for provision of road overbridges/under bridges and other safety
features at unmanned Railway Crossings.
• In order to mobilise the fund, the Central Road Fund Act 2000 proposed to levy and
collect by way of cess, a duty of excise and duty of customs on petrol and high speed

IoI Page 30
collect by way of cess, a duty of excise and duty of customs on petrol and high speed
diesel oil.
Learning: The bill seeks to amend the Central Road Fund Act, 2000, to allocate 2.5% of the
proceeds of CRF for development and maintenance of NWs and a reduction in the share
provided for development of National Highways.
http://www.insightsonindia.com/2017/07/25/insights-daily-current-affairs-25-july-2017/

58. The Banking Regulation (Amendment) Bill, 2017 seeks to address which of these major issues
in the banking sector?
A. Non-performing assets
B. Lack of competition in public sector banks
C. Increasing participation of women in top tier management of banks
D. Financial inclusion in rural areas

Correct Answer : A

Background:
• Stressed assets (NPAs or bad debt) in the banking system have reached unacceptably
high levels and hence, urgent measures were required for their speedy resolution.
Therefore, the government considered it necessary to make provisions in the Banking
Regulation Act, 1949 for authorizing the Reserve Bank of India to issue directions to any
banking company or banking companies to effectively use the provisions of the
Insolvency and Bankruptcy Code, 2016 for timely resolution of stressed assets.
Learning:
• The 2017 bill gives powers to the Reserve Bank of India (RBI) to ask any bank to initiate
insolvency proceedings and give directions for resolution of stressed assets. The RBI
could also be empowered to issue other directions for resolution, appoint or approve for
appointment, authorities or committees to advise the banking companies for stressed
asset resolution.
http://www.insightsonindia.com/2017/07/25/insights-daily-current-affairs-25-july-2017/

59. In 2015, under the Indradhanush plan, the government had announced capital infusion of
over seventy thousand crores in public sector banks for four years. However, credit rating
agencies had pointed out that the sum was insufficient because banks needed additional
funds to
1. Meet Basel-III norms
2. Tackle the menace of rising bad loans
Which of the above is/are correct?
A. 1 only
B. 2 only
C. Both 1 and 2
D. None

Correct Answer : C

Background:
• "Basel III" is a comprehensive set of reform measures, developed by the Basel Committee
on Banking Supervision, to strengthen the regulation, supervision and risk management
of the banking sector.
IoI Page 31
of the banking sector.
• It is an improvement over Base I and Basel II norms released much earlier.
• These norms may include reducing bank investment in risky assets, increasing their
capital base, strengthening management etc.
Statement 1:
• Rating agencies pointed that apart from PSBs face lack of competition and if business as
usual continues, they will face dearth of funds for expansion because: Private sector will
give them a tough competition in coming years Basel III norms required more funds to
absorb greater risks for banks
Statement 2: The background has been explained in another question on banking amendment
bill.
http://www.insightsonindia.com/2017/07/29/insights-daily-current-affairs-29-july-2017/

60. Methanol is a promising fuel for waterways in India because


1. It does not have any smog based emissions.
2. It is cheaper than most fossil fuels.
3. India manufactures all its requirement of methanol by domestic production.
Select the correct answer using the codes below.
A. 1 and 2 only
B. 2 and 3 only
C. 2 only
D. 3 only

Correct Answer : C

Justification:
• Across the world, methanol is emerging as a clean, sustainable transportation fuel of the
future. Methanol can be blended with gasoline in low-quantities and used in existing
road vehicles, or it can be used in high-proportion blends.
Statement 1:
• Methanol is a clean-burning fuel that produces fewer smog-causing emissions such as
sulphur oxides (SOx), nitrogen oxides (NOx) and particulate matter. This can improve air
quality and related human health issues.
Statement 2 and 3:
• It is clean, cheaper than fossil fuels and a good substitute for heavy fuels. India imports
methanol from Saudi Arabia and Iran at present. Technology is also being commercialized
to use methanol as a diesel substitute.
Learning:
• Methanol is most commonly produced on a commercial scale from natural gas. It can
also be produced from renewable sources such as biomass and recycled carbon dioxide.
As a high-octane vehicle fuel, methanol offers excellent acceleration and power. It also
improves vehicle efficiency. The government has asked the think-tank NITI Aayog to
study the automobile standards developed in China to use methanol as an alternative
fuel. Experts believe that Methanol economy will help India use its vast reserves of coal
while driving import substitution.
http://www.insightsonindia.com/2017/08/01/insights-daily-current-affairs-01 -aug-2017/

61. E-RaKAM is a first-of-its-kind initiative that


1. Leverages technology to connect farmers of the smallest villages to the biggest markets of

IoI Page 32
1. Leverages technology to connect farmers of the smallest villages to the biggest markets of
the world
2. Establishes grading and standardization centres for agricultural produce in the remotest
locations of the country to promote food processing
Which of the above is/are correct?
A. 1 only
B. 2 only
C. Both 1 and 2
D. None

Correct Answer : A

Justification: E-RaKAM is a digital initiative bringing together the farmers, FPOs, PSUs, civil
supplies and buyers on a single platform to ease the selling and buying process of agricultural
products. Under this initiative, e-RaKAM centres are being developed in a phased manner
throughout the country to facilitate farmers for online sale of their produce. The portal is a
joint initiative by state-run-auctioneer MSTC and Central Warehousing Corporation
arm CRWC.
Q Source: http://www.insightsonindia.com/2017/08/02/insights-daily-current-affairs-02-aug-2017/

62. Who is a Parliamentary secretary?


A. Assists a Minister in his or her duties
B. Heads the Lok Sabha secretariat
C. Discharges the functions of speaker when she is on leave
D. In charge of convening and summoning a session of the Parliament

Correct Answer : A

Learning:
• A Parliament Secretary is similar to a Minister of State who assists a Minister in his or her
duties. They come next to cabinet ministers and ministers of state in the hierarchy of
ministerial executive.
• Recently there was a controversy over the 21 AAP MLAs who were appointed
Parliamentary Secretaries to Ministers of the Delhi government.
• It raises the issue whether they should be considered as ministers, and if so, should the
cap of 15% for the size of council of ministers should apply to them.

63. Consider the following statements about Liquid nitrogen, that was in news for some time.
1. It starts to boil once it reaches room temperature.
2. It is an inflammable gas.
3. It is banned for use as an additive in frozen food in India.
Select the correct answer using the codes below.
A. 1 only
B. 2 and 3 only
C. 3 only
D. None of the above

Correct Answer : D

IoI Page 33
Justification: Statement 1:
• Liquid nitrogen, which has a boiling point of -195.8 degree Celsius, is used by molecular
gastronomy chefs to instantly freeze food and drinks. Recently, it is also being popular in
preparing cocktails, as it can immediately chills glasses of ingredients. It also adds a
smoking effect to the drink. It cools any substance immediately. The rapidity of cooling is
also responsible for smaller ice crystals which gives a smooth texture to the ice cream.
Issues: As it evaporates, liquid nitrogen freezes everything around it, including tissues
that come in contact with it.
• According to experts, food and drinks that are prepared with liquid nitrogen should be
consumed only after all the gas has bubbled off.
Statement 2:
• It is a coloured, odourless, non-inflammable gas. It is a cryogen, which means it is used to
create extremely low temperatures. The gas is so cold that it is capable of rapid freezing
any living tissue coming in contact with it.
Statement 3:
• Liquid nitrogen is permitted as an additive in frozen food as per the guidelines of the
national regulatory body Food Safety and Standards Authority of India (FSSAI).
• However, when it comes to the use of liquid nitrogen it is a grey area. There is no clear-
cut guideline for it and generally it is considered to be a novel technique, which can be
used by food business operators.
http://www.insightsonindia.com/2017/08/01/insights-daily-current-affairs-01 -aug-2017/

64. Under WTO rules, a member country cannot discriminate between its trade partners. Hence
1. A country cannot enter into a free trade agreement with another country without the
authorization of WTO
2. If a special status is granted to a trade partner, it should usually be extended to all
members of the WTO.
Which of the above is/are correct?
A. 1 only
B. 2 only
C. Both 1 and 2
D. None

Correct Answer : B

Justification: Statement 1:
• Countries don’t require the permission of WTO for their trade related activities.
However, they can move WTO if a country violates the rules of trade set by WTO. They
are free to enter into FTAs with other countries.
Statement 2: This is called as MFN status. As per a WTO rule, every member of WTO requires
to accord this status to other member countries. India has already granted this status to all
WTO members including Pakistan. Pakistan is yet to reciprocate. Most Favoured Nation is a
treatment accorded to a trade partner to ensure nondiscriminatory trade between two
countries vis-a-vis other trade partners. MFN, however, allows for countries to enter into
regional trade agreements. It also relaxes such requirements in case of LDCs and other
developing countries.
http://www.insightsonindia.com/2017/08/01/insights-daily-current-affairs-01 -aug-2017/

IoI Page 34
65. The objective of the GST composition scheme is to
1. Reduce duties and taxes on all raw materials to zero
2. Maximize the tax on value added to the final product
3. Reduce the tax compliance cost for small businesses
4. Charge differential taxes for the same product based on different compositions
Select the correct answer using the codes below.
A. 3 only
B. 1 and 2 only
C. 4 only
D. 1, 2 and 3 only

Correct Answer : A

Justification:
• The composition scheme is an alternative method of levy of tax designed for small
taxpayers whose turnover is up to Rs 75 lakh — Rs 50 lakh in the case of eight north-
eastern states and the hilly state of Himachal Pradesh. To promote small business, and
reduce their tax burden, the scheme provides that: While a regular taxpayer has to pay
taxes on a monthly basis, a composition supplier is required to file only one return and
pay taxes on a quarterly basis. Also, a composition taxpayer is not required to keep
detailed records that a normal taxpayer is supposed to maintain.
Learning:
• As per the Central GST Act, businesses are eligible to opt for the composition scheme if a
person is not engaged in any inter-state outward supplies of goods and not into making
any supply of goods through an electronic commerce operator who is required to collect
tax at source.
• Five lakh businesses have opted for the GST composition scheme, which allows them to
pay taxes at a concessional rate and makes compliance easy.
Http://www.insightsonindia.com/2017/08/01/insights-daily-current-affairs-01 -aug-2017/

66. A new representative body — US- India Strategic Partnership Forum (USISPF) — is being set
up to further enhance
A. Business relations between the two countries
B. Defense related strategic relations
C. Education and technology related exchange
D. All of the above

Correct Answer : A

Learning:
• The non-profit corporation aims to promote bilateral trade and work on creating
meaningful opportunities that have the power to change the lives of citizens.
• The body will work closely together with businesses and government leaders to achieve
its goals of driving economic growth, job creation, innovation, inclusion and
entrepreneurship.
http://www.insightsonindia.com/2017/08/02/insights-daily-current-affairs-02-aug-2017/

67. The Ministry of Tribal Affairs is implementing the “Scheme of Development of Particularly
IoI Page 35
67. The Ministry of Tribal Affairs is implementing the “Scheme of Development of Particularly
Vulnerable Tribal Groups(PVTGs)”. The scheme focuses on which of these key parameters of
development?
1. Providing regular unemployment allowances
2. Economic development of PVTGs through horticulture
3. Land distribution and land development
4. Promotion of traditional and tribal games and sports
Select the correct answer using the codes below.
A. 2, 3 and 4 only
B. 1 and 3 only
C. 2 and 4 only
D. 1, 2, 3 and 4

Correct Answer : A

Justification:
• It aims at planning their socio-economic development in a comprehensive manner while
retaining the culture and heritage of the community.
• Activities under the Scheme includes Livelihood, Employment opportunities and
economic development of PVTGs through Agriculture, Horticulture, Animal Husbandry,
Dairy, and Skilling/ Vocational Training etc. It further includes Education, Health (Gap
filling for effective health service delivery beyond NHM etc) and Provision of safe drinking
water. Land distribution, land development, Social security, Connectivity (Road and
Telecommunication), Supply of Electricity and irrigation are also taken care of including
sports.
• Housing and Habitat covers funds for housing for PVTGs, primarily through special
assistance under IAY and additionally through this scheme, for gap filling and for
enhancing scope of coverage.

68. A microbial consortium called SONA was recently seen in news. It is related to
A. Bio-remediation of oil spills
B. Identifying gold mines using microbes without physical exploration
C. Improving jute cultivation yield
D. Treatment of sewage sites

Correct Answer : C

Learning: Improved Cultivation and Advanced Retting Exercise for Jute (Jute – ICARE) was
launched in 2015 to popularize/introduce some of the better agronomic practices and recently

developed microbial-assisted retting among farmers. Central Research Institute for Research in
Jute and Allied Fibres (CRIJAF) has also recently developed a microbial consortium called
SONA, to enhance the quantity of fibre yield by 20% as well as its quality in terms of grade by
at least 1 ½ grades. Also, under the project, regular SMSes are sent in regional languages on
improved practices in jute cultivation, to registered farmers.

69. The Union Cabinet has given its ex-post facto approval for amendment of the Constitution
(Application to Jammu & Kashmir) Order, 1954 by way of the Constitution (Application to
IoI Page 36
(Application to Jammu & Kashmir) Order, 1954 by way of the Constitution (Application to
Jammu & Kashmir) Amendment Order, 2017. This was done in order to
A. Apply GST regime in the State of Jammu & Kashmir
B. Allow alternative communication models like prepaid sim cards in J&K that were earlier
blocked due to security concerns
C. Provide a separate flag to the state of J&K
D. Assign a permanent advisor from the Union government to the state of J&K

Correct Answer : A

Learning: An ordinary amendment to the constitution does not apply necessarily to the state
of J&K which has a separate constitution of its own. The President needs to give approval to
certain amendments about which you will read later in Laxmikanth. The approval paves the
way for applicability of Goods and Services Tax regime in the State of Jammu & Kashmir.

70. The Indian Council for Cultural Relations (ICCR has several Indian Cultural Centres abroad.
Consider the following about them.
1. These cultural centres showcase India’s soft power.
2. There is no Indian Cultural Centre in Central Asia and the Middle east.
3. Activities in these centres are organized with the help of involvement of local
community and Indian Diaspora.
Select the correct answer using the codes below.
A. 1 and 3 only
B. 2 only
C. 2 and 3 only
D. 1 only

Correct Answer : A

Justification: Statement 1: Soft power of India includes its culture, diaspora, ancient heritage
like yoga and the skills of its population.
Statement 2: During the recent visit of Prime Minister of India to Israel, announcement for the

opening of Indian Cultural Centre in Israel was made. There are centres in Central Asian
countries like Kazakhstan. Russia is another centre, but it is not counted in central Asia.
Statement 3: Cultural Centres overseas are ICCR’s important arms, which play significant role
in promotion of India’s soft power strength through its variety of activities like facilitating
teaching of Indian dance, music, yoga, Hindi etc., organizing cultural performances etc.

71. The Amasr Act, 2010 that was recently seen in news concerns
A. Preservation of monuments
B. Immigration and refugees in India
C. Genetic cleansing of livestock population
D. Prohibited chemicals and weapons

Correct Answer : A

IoI Page 37
Learning: The proposal for amendment of the Amasr Act, 2010 has been approved by the
Cabinet and the pertaining Bill has been moved to Lok Sabha. The proposal is to allow public
works or projects essential for public in prohibited area within 100 meter from protected
monument but not having substantial impact on preservation, safety, security or access to the
monument or its immediate surrounding including visual ambiance.

72. Private Entrepreneurs Guarantee (PEG) Scheme concerns


A. Manufacturing MSMEs
B. Warehousing capacity for food storage
C. Venture Capital Funds that have been established by the Government
D. Guaranteed Land acquisition for mega projects

Correct Answer : B

Learning: Under PEG, storage capacity is created by private parties, Central Warehousing
Corporation (CWC) and State Agencies for guaranteed hiring by FCI. This is because the state
alone cannot provide such facilities at all locations. It makes economic sense for private
players to finance the construction of these warehouses and then lease it out for rent.
Q Source: www.dfpd.nic.in/peg.htm

73. Which of these nations share closest latitudes?


#46460
A. Aswan, Riyadh and Abu Dhabi
B. Cairo, Istanbul and Damascus
C. Tehran, Ashgabat and Aleppo
D. Mosul, Muscat and Tel-Aviv

Correct Answer : A

Justification: Closest latitudes mean that they are located in a narrow band of latitude. They
can be located at different longitudes.
Q Source: Map based questions: West Asia

74. Consider the following with reference to rubber production in India.


1. Kerala is the largest producer of natural rubber in India.
2. The government has deregulated synthetic rubber industry in India.
3. As of date, no dedicated scheme or policy has been enacted to develop rubber sector in
India.
Select the correct answer using the codes below.
A. 1 only
B. 2 and 3 only
C. 1 and 2 only
D. 1, 2 and 3

Correct Answer : C

Justification: Statement 1: India is the third largest natural rubber producing country of the

IoI Page 38
Justification: Statement 1: India is the third largest natural rubber producing country of the
world, next to Thailand and Indonesia, producing about 9 per cent of the global output. Kerala
is the largest producer, followed by TN.
Statement 2: So far as artificial/synthetic rubber is concerned, Chemical & Petrochemical
Industry is delicensed and decontrolled and Government acts as facilitator in this sector.
Statement 3: In order to increase production of Natural Rubber in the country the
Government through the Rubber Board is implementing the scheme “Sustainable and Inclusive
Development of Natural Rubber Sector” wherein support is provided for plantation
development and extension, strengthening research, technology upgradation and market
development, Human resource development etc.

75. Consider the following matches of periods with the beginning of major activities in the history
of human civilization:
1. Building of megaliths: 3000 years ago
2. Cotton cultivation: 2000 years ago
3. Domestication of animals: 12000 years ago
Select the correct answer using the codes below.
A. 1 and 2 only
B. 1 and 3 only
C. 2 only
D. 1, 2 and 3

Correct Answer : B

Justification: Statement 1: Beginning of cities started about 4700 years ago. Settlement at
Inamgaon began between 3600 and 2700 years ago.
Statement 2: This started at Mehrgarh about 7000 years ago.
Statement 3: Domestication was a gradual process that took place in many parts of the world.

It began about 12,000 years ago. Some of the earliest plants to be domesticated were wheat
and barley. The earliest domesticated animals include sheep and goat.

Q Source: General: Chapter 1: Themes in Indian History: Part -I

76. Which of these strive to ensure equity among Indian citizens?


1. Constitution of India
2. Laws enacted by the Parliament
3. Schemes and programmes of the government
Select the correct answer using the codes below.
A. 1 only
B. 2 and 3 only
C. 1 and 3 only
D. 1, 2 and 3

Correct Answer : D

Justification: Statement 1: Fundamental rights ensure right to equality and remove arbitrary
discrimination. DPSP strives to create a equitable society through special provisions for weaker

IoI Page 39
discrimination. DPSP strives to create a equitable society through special provisions for weaker

sections.
Statement 2: Laws like SC/ST Atrocities act, Forest Rights Act ensure justice to weaker and
marginalized sections.
Statement 3: MGNREGA, Food security Act etc. ensure that left behind sections are provided
with livelihood, food etc.

Q Source: Page 10: Social and Political Life: 7th NCERT

77. The first state in India to introduce the Mid Day Meal (MDM) scheme was
A. Kerala
B. Gujarat
C. Tamilnadu
D. Bihar

Correct Answer : C

Learning: Other than the TN model of MDM, in 2001, the Supreme Court asked all state
governments to begin this programme in their schools within six months. Later, it was made a
central programme that today feeds more than 12 crore children all over India. The Midday
Meal Scheme is covered by the National Food Security Act, 2013. The central and state
governments share the cost of the Midday Meal Scheme, with the centre providing 60 percent
and the states 40 percent

Q Source: Page 11: Social and Political Life: 7th NCERT

78. The United Nations Children's Fund (UNICEF) is a United Nations (UN) programme that
provides humanitarian and developmental assistance to children and mothers in developing
countries. Consider the following with reference to UNICEF’s association with India.
1. UNICEF provided equipment and technical assistance for the first Penicillin Plant
established in India.
2. Within a decade of independence, UNICEF signed an agreement with the Government
of India to fund some cooperative milk processing plants to support white revolution.
3. UNICEF helped India in developing the India Mark II which is now the world’s most
widely used hand pump.
Select the correct answer using the codes below.
A. 1 only
B. 2 only
C. 3 only
D. 1, 2 and 3

Correct Answer : D

Justification: Statement 3: During the 1970s, UNICEF became a key partner with the
government of India in the world’s largest rural water supply programme.
UNICEF brought drilling rigs to India which could drill boreholes in hard rock. The Government
supplied handpumps. But, due to some problems, India mark II was later adopted.
Learning: Gender issues were mainstreamed into the training and communication strategy for
IoI Page 40
Learning: Gender issues were mainstreamed into the training and communication strategy for
the 2011 Census with the help of UNESCO. The Government, in partnership with UNICEF,
WHO, the Bill & Melinda Gates Foundation, Rotary International and the Centers for Disease
Control and Prevention contributed to almost universal awareness of the need to vaccinate all
children under five against polio. As a result of these efforts, India was removed from the list
of endemic countries in 2014.
Q Source: Additional Research: Page 25: Social and Political Life: 7th NCERT
http://unicef.in/WhoWeAre/History

79. Archaeological Survey of India was founded by


A. Alexander Cunningham
B. Joseph Barruel
C. Lord Lawrence
D. Mortimer Wheeler

Correct Answer : A

Learning: ASI is attached to the Ministry of Culture that is responsible for archaeological
research and the conservation and preservation of cultural monuments in the country.
Cunningham carried out a detailed survey of the Buddhist monuments which lasted for over
half a century. While he funded many of his early excavations himself, in the long run, he
realised the need for a permanent body to oversee archaeological excavations

Q Source: Additional Research: Page 6: Themes in Indian History – Part I

80. Which of these sovereign states do NOT have armed forces?


1. Costa Rica
2. Solomon Islands
3. Madagascar
Select the correct answer using the codes below.
A. 1 and 2 only
B. 3 only
C. 1 and 3 only
D. 2 only

Correct Answer : A

Justification: There are 23 countries that have no active military force, including Costa Rica,
Iceland, Panama, Micronesia, the Marshall Islands, and The Vatican. These nations vary in size,

history, and reasons behind choosing to not have a standing army.


Statement 1: Not having an army helped the Costa Rican government to spend the money that
the army would have used, on health, education and other basic needs of the people.

Q Source: Additional Research: Page 28: Social and Political Life: 7th NCERT

81. That the State should “raise the level of nutrition and the standard of living and to improve
public health” is a provision under

IoI Page 41
public health” is a provision under
A. Fundamental right
B. Directive Principle of State policy
C. Fundamental Duty
D. Legal rights

Correct Answer : B

Learning: Right to Health is not included as an explicit fundamental right in the Indian
Constitution. Most provisions related to health are under DPSP.
These are:
• Article 38 says that the state will secure a social order for the promotion of welfare of the
people. Providing affordable healthcare is one of the ways to promote welfare. Article
39(e) calls the state to make sure that health and strength of workers, men and women,
and the tender age of children are not abused.
• Article 41 imposes duty on state to provide public assistance in cases of unemployment,
old age, sickness and disablement etc.
• Article 42 makes provision to protect the health of infant and mother by maternity
benefit.
• Article 47 make it duty of the state to improve public health, securing of justice, human
condition of works, extension of sickness, old age, disablement and maternity benefits
and also contemplated. Further, State’s duty includes prohibition of consumption of
intoxicating drinking and drugs are injurious to health.
• Article 48A ensures that State shall Endeavour to protect and impose the pollution free
environment for good health.

Q Source: Page 29: Social and Political Life: 7th NCERT

82. Which of these nations has nationalized the Suez Canal?


A. Egypt
B. Syria
C. Israel
D. Saudi Arabia

Correct Answer : A

Learning: The Suez Canal is considered to be the shortest link between the east and the west
due to its unique geographic location. It is an important international navigation canal linking
between the Mediterranean sea at Port said and the red sea at Suez. Egypt nationalized the
canal in 1956.
Q Source: Map based questions: West Asia

83. Consider the following statements.


1. A Governor is the Head of a State of India, whereas President is the head of the Indian
state.
2. Governors are appointed by the President of India and he is free to consult the state chief
minister in this matter.
Which of the above is/are correct?
A. 1 only
IoI Page 42
A. 1 only
B. 2 only
C. Both 1 and 2
D. None

Correct Answer : C
Justification: Statement 1:
• The Head (ceremonial) of the State is the Governor. Head of government is the Chief
Minister.
• She/He is appointed by the Central Government to ensure that the State Government
works within the rules and regulations of the Constitution.
Statement 2:
• The President may nor may not consult the CM, but it is a convention to do so since both
need to work in tandem.

Q Source: Page 33: Social and Political Life: 7th NCERT

84. An Internal Complaints Committee (ICC) is usually constituted at workplaces according to


which of these acts?
A. Consumer Protection Act, 1984
B. Factories Act, 1948
C. Sexual Harassment of Women at Workplace (Prevention, Prohibition & Redressal) Act,
2013
D. None of the above

Correct Answer : C
Learning:
• Under the Act, which also covers students in schools and colleges as well as patients in
hospitals, employers and local authorities will have to set up grievance committees to
investigate all complaints.
• These grievance committees are known as ICC.
• The District Officer is required to constitute a Local Complaints Committee at each
district, and if required at the block level.
• The Complaints Committees have the powers of civil courts for gathering evidence.

Q Source: Additional Research: Page 64: Social and Political Life: 7th NCERT

The Advertising Standards Council of India (ASCI) is a


1. Self-regulatory institution
2. Non-government body
3. Body that can prohibit certain ads from appearing on TV
Select the correct answer using the codes below.
A. 1 and 2 only
B. 3 only
C. 2 only
D. 1, 2 and 3

Correct Answer : D

IoI Page 43
Concept & Background:
• When an advertiser is creating an ad, the consumer is his audience.
• If a consumer feels that a particular advertisement is in bad taste or is false in its claims,
they need a body or council to whom they can air their grievances and who will take any
appropriate action, if necessary.
• ASCI as a self-regulatory body governing advertising content is the ideal medium as its
purpose is to serve both the advertisers as well as the consumers. There is no other non
governmental body in India which regulates the advertising content that is released in
India.
Justification:
• If an ad that is released in India seems objectionable, a person can write to ASCI with
their complaint.
• This complaint will be deliberated on by the complains council after providing due
process to advertiser to defend the ad against the complaint. If complaint is upheld then
the ad is voluntarily either withdrawn or modified.
• In 2007, the Government of India amended the Cable TV Network Rules’ Advertising
Code by which ads which violate ASCI code cannot be permitted on TV.

Q Source: Additional Research: Page 83: Social and Political Life: 7th NCERT

85. Which of these nations border both Caspian Sea and Black Sea?
A. Iran
B. Turkey
C. Russia
D. Georgia

Correct Answer : C

Q Source: Map based questions: West Asia

86. Consider the following statements.


1. Every state in India has a Legislative Assembly.
2. The strength of legislative assembly in a state is fixed by the President.

Which of the above is/are correct?


A. 1 only
B. 2 only
C. Both 1 and 2
D. None

Correct Answer : A
Justification: Statement 1:
• But, all UTs do not have a legislative assembly. Only, Delhi and Puducherry have it.
Statement 2:
• This is based on the population of the state. A state with a larger population like UPwill
have more seats in the assembly.

Q Source: Page 32: Social and Political Life: 7th NCERT

IoI Page 44
87. Nations that border the Persian Gulf are
1. Saudi Arabia
2. United Arab Emirates
3. Iran
4. Kuwait
5. Jordan
Select the correct answer using the codes below.
A. 2 and 4 only
B. 1, 2, 3 and 4 only
C. 3 and 5 only
D. 1 and 5 only

Correct Answer : B

Q Source: Map based questions: West Asia

88. What are the properties of X-rays?


1. X-rays have much longer wavelengths than visible light.
2. X-ray photons carry enough energy to ionize atoms and disrupt molecular bonds.
3. X-rays do not interact with matter at all.
Select the correct answer using the codes below.
A. 1 and 2 only
B. 2 only
C. 3 only
D. 1, 2 and 3

Correct Answer : B
Justification: Statement 1:
• They have shorter wavelengths and higher frequencies. This makes it possible to probe
structures much smaller than can be seen using a normal microscope.
• This property is used in X-ray microscopy to acquire high resolution images, and also in X-
ray crystallography to determine the positions of atoms in crystals.
Statement 2:
• This makes it a type of ionizing radiation, and therefore harmful to living tissue. A very
high radiation dose over a short period of time causes radiation sickness, while lower
doses can give an increased risk of radiation-induced cancer.
Statement 3:
• X-rays interact with matter in three main ways, through photoabsorption, Compton
scattering, and Rayleigh scattering.
• The strength of these interactions depends on the energy of the X-rays and the elemental
composition of the material. We will try covering these separately in upcoming tests
because they are important.

Q Source: Additional Research: Page 25: Social and Political Life: 7th NCERT

89. Consider the following statements.


1. Dasht-e Kavir is a great salt desert located in Arabian Peninsula.
2. Dasht-e Lut is a large desert located in Iran that has been inscribed on UNESCO's World

IoI Page 45
2. Dasht-e Lut is a large desert located in Iran that has been inscribed on UNESCO's World
Heritage List.
Which of the above is/are correct?
A. 1 only
B. 2 only
C. Both 1 and 2
D. None

Correct Answer : B
Justification: Statement 1:
• Also known as Kavir-e Namak (literally 'salty lowlands') and the Great Salt Desert, it is a
large desert lying in the middle of the Iranian plateau.
Statement 2:
• The Lut Desert, widely referred to as Dasht-e Lut is a large salt desert located in the
provinces of Kerman and Sistan and Baluchestan, Iran. It is the world's 27th-largest
desert, and was inscribed on UNESCO's World Heritage List in 2016.
• The surface of its sand has been measured at temperatures as high as 70 °C making it one
of the world's driest and hottest places.

Q Source: Map based questions: West Asia

90. Harappans were aware of and practiced which of these number systems in their weights and
measures?
A. Binary system
B. Decimal system
C. Hexadecimal system
D. Both (a) and (b)
Correct Answer : D
Learning:
• Exchanges were regulated by a precise system of weights, usually made of a stone called
chert and generally cubical with no markings.
• The lower denominations of weights were binary (1, 2, 4, 8, 16, 32, etc). While the higher
denominations followed the decimal system, the smaller weights were probably used for
weighing jewellery and beads and bigger weights were used for food grains.

Q Source: Additional Research: Page 15: Themes in Indian History – I

91. Among the earliest chalcolithic cultures in India, the Ahar or Banas culture was discovered in
the
A. Mewar region of Rajasthan
B. Gwalior region of Madhya Pradesh
C. Bundelkhand region of Uttar Pradesh
D. Kangra region of Himachal Pradesh
Correct Answer : A
Learning:
• Nearly one hundred sites of the culture have been located along its principal axis, i.e., the
valleys of river Banas and its tributaries and subtributaries in Banswara, Udaipur etc.
• The technology at Ahar was based mainly on copper and very few microblades and

IoI Page 46
• The technology at Ahar was based mainly on copper and very few microblades and
microliths have been discovered.
• Ahars were a separate culture from GJs. Located in northeast Rajasthan, the Ganeshwar-
Jodhpura complex which was an early centre of agriculture and copper metallurgy in the
subcontinent.

Q Source: Additional Research: Page 12: Themes in Indian History – I

92. Consider the following statements.


1. Most Ashokan inscriptions were in the Greek language while those in the northwest of
India subcontinent were in Aramaic and Prakrit.
2. Ashokan inscriptions were written in both Prakrit and Brahmi scripts.
Which of the above is/are correct?
A. 1 only
B. 2 only
C. Both 1 and 2
D. None
Correct Answer : B
Justification: Statement 1:
• A large number of them were in Prakrit, but in the NW direction one could find Aramaic
and Greek languages.
Statement 2:
• The Aramaic and Greek scripts were used for inscriptions in Afghanistan. James Prinsep,
an officer in the mint of the East India Company, deciphered Brahmi and Kharosthi, two
scripts used in the earliest inscriptions and coins. This gave a new direction to
investigations into early Indian political history.

Q Source: Page 28: Themes in Indian History – I


93. With reference to Matsays, one of the early states, consider the following statements.
1. They were one of the Indo-Aryan tribes of Vedic India.
2. Their rule extended to the whole of eastern and central India.
3. They were an appendage of Ashmaka dynasty.
Select the correct answer using the codes below.
A. 1 only
B. 2 and 3 only
C. 3 only
D. 1, 2 and 3
Correct Answer : A
Justification: Statement 1 and 2:
• By the late Vedic period, they ruled a kingdom located south of the Kurus, and west of
the Yamuna River which separated it from the kingdom of the Panchalas. It roughly
corresponded to the former state of Jaipur in Rajasthan.
Statement 3:
• The capital of Matsya was at Viratanagari (present-day Bairat). Alwar has been a part of
Matsya region. Ahmakas were further down south, see map below

94. In Ancient India, a means of claiming high status for the Kings was to identify with a variety of
deities. Consider the following with reference to it.

IoI Page 47
deities. Consider the following with reference to it.
1. Kushana rulers installed god-like colossal statues of them.
2. Some Kushanas adopted the title devaputra which implied the “son of the God”.
3. Vijayanagara kings claimed to rule on behalf of the god Virupaksha.
4. Vijayanagara rulers adopted the title ‘brahma-varna” to segment themselves as the
highest castes equivalent to God.

Select the correct answer using the codes below.


A. 1, 2 and 3 only
B. 2 and 4 only
C. 3 only
D. 1, 2, 3 and 4
Correct Answer : A
Justification: Statement 1:
• Huge statues of Kushanas have been found installed in a shrine at Mat near Mathura
(Uttar Pradesh). Similar statues have been found in a shrine in Afghanistan as well.
Statement 2:
• The title devaputra was possibly inspired by Chinese rulers who called themselves sons of
heaven.
Statement 3:
• They indicated their close links with the gods by using the title “Hindu Suratrana”
meaning Hindu Sultan.

Q Source: Additional Research: Page 36: Themes in Indian History – I

95. The institution of Samanta finds mention for the first time in epigraphs of northern India
dating to the 6th century. It is most closely associated with the origin and growth of which of
these systems in India?
A. Feudalism
B. Bonded Labour
C. State ownership of land
D. Standing army system

Correct Answer : A
Justification:
• Options B, C and D existed much prior to 6th century, so they can’t be the answer.
• The institution is considered to belong properly to the Gupta Empire, however, some
vague evidence is found in South India as well.
• The Samanta in South-India was used to mean a vassal to an emperor. In North-India, the
earliest use of the term in a similar sense was in Bengal where someone was described as
Samanta-Chudamanih (best among feudatories) of the imperial Guptas.
• The Samanta vassal provided military support to the Monarch and governed over a
portion of a territory.
Learning:
• The term Indian feudalism is used to describe zamindar, jagirdar, sardar, mankari,
deshmukh, chaudhary and samanta.
• In this system, a nobility gave the emperor military services in exchange of land. There
was a separate class of labourers or peasants who were provided military protection in
exchange of labour and share of their produce. They were allowed to live on the lord’s

IoI Page 48
exchange of labour and share of their produce. They were allowed to live on the lord’s
land.

Q Source: Additional Research: Themes in Indian History – I

96. In the Indian subcontinent, the first coins to bear the names and images of rulers were issued
by the
A. Indo - Greeks
B. Mauryas
C. Parthians
D. Yaudheyas

Correct Answer : A
Learning:
• Mauryas issued punch marked coins made of silver and copper. Indo-greeks used names
of rulers on coins. First gold coins were issued by Kushanas (some sources dispute this
fact).
• Coins were also issued by tribal republics of Punjab and Haryana called the Yaudheyas.
• Archaeologists have unearthed several thousand copper coins issued by the Yaudheyas.
• Some of the most spectacular gold coins were issued by the Gupta rulers.

Q Source: Page 44: Themes in Indian History – I

97. The earliest inscriptions recording royal land grants were issued during
A. Guptas
B. Satvahanas
C. Pallavas
D. Hoysalas

Correct Answer : B
Justification :
• In early India, land possessed great value and the gift of an estate was a marker of status.

• While the earliest inscriptions recording royal land grants were issued during Satavahana
rule, the practice truly grew in scope from the fourth century CE.
• By the 5th-6th centuries, ruling dynasties across the subcontinent, as well as their
subordinates and feudatories were all engaged in making such grants. Land grant
charters were usually engraved on sheets of hammered copper. They are described as
tamrapatta, a ‘plate of copper’ or tamrashasana, ‘an order in copper’.

Q Source: Page 50: Themes in Indian History – I

98. What is a catcher beach?


A. A beach with low shorelines
B. A tidal estuary with no opening mouth
C. A place where marine debris tends to pile up or aggregate
D. A beach devoid of mangroves and corals

IoI Page 49
Correct Answer : C
Justification :Learning:
• A catcher beach should not be confused with a dumping ground or heavily trashed public
beach.
• A catcher beach typically receives its accumulations of debris due to its shape and
location in combination with high-energy waves, storms, or winds.
• A specific example of a catcher beach can be found along the shores of Gore Point,
Alaska. The geography of this location makes it a very high-density catcher beach, as it
sticks out like a hook into the Gulf of Alaska current.

Q Source: Surprise questions

100. Centre has banned the manufacture and import of Polychlorinated Biphenyls (PCBs).
This is because
A. It is a toxic pollutant.
B. It is responsible for coral bleaches across the eastern coast of India.
C. It was being oversupplied in the market leading to price crashes and poor producer
margins.
D. It was heavily used by farmers as an additive in their NPK fertilizers.

Justification : Learning:
• PCBs are synthetic organic chemicals used in many different products including electrical
equipment, inks, adhesives, flame-retardants, and paints. Its production and use are now
severely restricted in many countries because of possible impacts on human health and
the environment.
• The use of PCBs in any form shall be completely prohibited by 2025.
• The government has directed users not to drain or discharge PCBs directly or indirectly
on land, in surface water or from effluent treatment plants.
• The government's decision is in line with the Stockholm Convention, under which
signatory countries are to prohibit or take necessary legal and administrative measures
to eliminate the production and use of PCBs.

IoI Page 50

Вам также может понравиться